LSAT and Law School Admissions Forum

Get expert LSAT preparation and law school admissions advice from PowerScore Test Preparation.

 Administrator
PowerScore Staff
  • PowerScore Staff
  • Posts: 8917
  • Joined: Feb 02, 2011
|
#59072
Please post your questions below!
 AWash180
  • Posts: 6
  • Joined: Oct 03, 2018
|
#59130
I choose the correct answer for this question. From what I recall I skipped this question then came back to it when I could reference all of my past work. For questions like this one do you have any suggestions on how to find the answer without having to test each answer choice?
 Rachael Wilkenfeld
PowerScore Staff
  • PowerScore Staff
  • Posts: 1358
  • Joined: Dec 15, 2011
|
#59455
Hi Awash,

Great question.

Whenever possible on local questions, I try to draw out a mini-diagram. For this one, we know that R is in first, which puts T in 7th. It looks like there's a large open section in the middle, but we can think about where we see limitations. The first place I would look is at where the h/s block could go. Either it's going to go in 2/5 or 3/6. Which could look like this:

Scenario 1: R S P L H M T

or

Scenario 2: R L H M P S T

or

Scenario 3: R _ S _ _ H T

With those three in mind, we can look at the answers.

Answer choice (A): The third sale is H. In all three, I know where H is. If I know H is in 3, I can fill out the rest of scenario 2 above. This is the correct answer.

Answer choice (B): The third sale is S. In the final situation, S is third, but we don't know where several variables go. This cannot be the correct answer.

Answer choice (C): The fourth sale is L. In the final situation, L can be 4th and in the first situation L is in 4th. It could be either situation. This can't be the right answer.

Answer choice (D): The fourth sale is on M. This could occur in either scenario 2 or 3 above. Since we don't know which it cannot be the right answer.

Answer choice (E): The sixth sale is on H. This is only in the final scenario, but there are still blank spaces. I don't know how those must be filled, so this is not the correct answer.

Hope that helps!
Rachael
 lsathelppls
  • Posts: 5
  • Joined: Nov 12, 2018
|
#60359
why can't h/s go in spots 2/6?

is there something wrong with it being R S P M L H T ?
 gcs4v333
  • Posts: 19
  • Joined: Oct 09, 2018
|
#60460
lsathelppls wrote:why can't h/s go in spots 2/6?

is there something wrong with it being R S P M L H T ?
I'd like to second this question.
 Malila Robinson
PowerScore Staff
  • PowerScore Staff
  • Posts: 296
  • Joined: Feb 01, 2018
|
#60463
Hi lsathelppls & gcs4v333,
The problem is that the question requires your chart to be absolutely determined. However, putting h/s in 2/6 would leave m/l switchable in spots 4/5 it could be ml or lm, so your chart is not absolutely determined.
Hope that helps!
-Malila

Get the most out of your LSAT Prep Plus subscription.

Analyze and track your performance with our Testing and Analytics Package.